- PowerScore Staff
- Posts: 5972
- Joined: Mar 25, 2011
- Tue Dec 07, 2010 7:26 pm
#92490
Complete Question Explanation
Justify the Conclusion. The correct answer choice is (E)
The argument is given in premise-conclusion-premise form, and the conclusion is that "the city needs to purchase new subway cars." The word "so" often introduces a conclusion, and the word "since" often introduces a premise. The argument itself is weak since there may be other ways to have an efficient attractive subway system other than by purchasing new subways cars (for instance, the city could renovate the existing system).
The question stem asks you to find the statement that, when combined with the premises, will make the conclusion a logical conclusion. From this perspective, when the correct answer choice is added to the premises, the conclusion should automatically follow. It is therefore important to establish that the correct answer choice will produce the conclusion that new subway cars need to be purchased by the city.
Answer Choice (A): Incorrect. This answer choice suggests that the city should do something to improve the subway system, but it does not specify that the city must purchase new subway cars.
Answer Choice (B): Incorrect. This answer is incorrect because it addresses "cost-effective subway cars," whereas the conclusion discusses "new subway cars."
Answer Choice (C): Incorrect. The conclusion is clear that the city "needs" to purchase new subway cars. Answer choice (C) simply states that subway cars make better economic sense than "many of the other investment options open to the city"; this does not produce the conclusion that the city must purchase new subway cars but rather that purchasing such cars was a reasonable decision.
Answer Choice (D): Incorrect. Affordability is not the issue in the stimulus.
Answer Choice (E): This is the correct answer choice. This answer choice can be diagrammed using conditional arrows, as follows:
According to the premises in the stimulus, having an efficient and attractive subway system makes good economic sense, and the city always does what makes good economic sense. Therefore, the city will strive to have an efficient and attractive subway system. Combining this with answer choice (E), it follows that the city needs new subway cars. Since this matches the conclusion in the stimulus, answer choice (E) must be correct.
Justify the Conclusion. The correct answer choice is (E)
The argument is given in premise-conclusion-premise form, and the conclusion is that "the city needs to purchase new subway cars." The word "so" often introduces a conclusion, and the word "since" often introduces a premise. The argument itself is weak since there may be other ways to have an efficient attractive subway system other than by purchasing new subways cars (for instance, the city could renovate the existing system).
The question stem asks you to find the statement that, when combined with the premises, will make the conclusion a logical conclusion. From this perspective, when the correct answer choice is added to the premises, the conclusion should automatically follow. It is therefore important to establish that the correct answer choice will produce the conclusion that new subway cars need to be purchased by the city.
Answer Choice (A): Incorrect. This answer choice suggests that the city should do something to improve the subway system, but it does not specify that the city must purchase new subway cars.
Answer Choice (B): Incorrect. This answer is incorrect because it addresses "cost-effective subway cars," whereas the conclusion discusses "new subway cars."
Answer Choice (C): Incorrect. The conclusion is clear that the city "needs" to purchase new subway cars. Answer choice (C) simply states that subway cars make better economic sense than "many of the other investment options open to the city"; this does not produce the conclusion that the city must purchase new subway cars but rather that purchasing such cars was a reasonable decision.
Answer Choice (D): Incorrect. Affordability is not the issue in the stimulus.
Answer Choice (E): This is the correct answer choice. This answer choice can be diagrammed using conditional arrows, as follows:
- Efficient and attractive subway system New subway cars
According to the premises in the stimulus, having an efficient and attractive subway system makes good economic sense, and the city always does what makes good economic sense. Therefore, the city will strive to have an efficient and attractive subway system. Combining this with answer choice (E), it follows that the city needs new subway cars. Since this matches the conclusion in the stimulus, answer choice (E) must be correct.
Dave Killoran
PowerScore Test Preparation
Follow me on X/Twitter at http://twitter.com/DaveKilloran
My LSAT Articles: http://blog.powerscore.com/lsat/author/dave-killoran
PowerScore Podcast: http://www.powerscore.com/lsat/podcast/
PowerScore Test Preparation
Follow me on X/Twitter at http://twitter.com/DaveKilloran
My LSAT Articles: http://blog.powerscore.com/lsat/author/dave-killoran
PowerScore Podcast: http://www.powerscore.com/lsat/podcast/